\documentclass[a5paper,10pt]{article} \usepackage{myXsim} \usepackage{tasks} % Title Page \title{DM2 \hfill EYRAUD Cynthia} \tribe{TST} \date{\hfillÀ render pour le Mercredi 24 février} \xsimsetup{ solution/print = false } \begin{document} \maketitle \begin{exercise}[subtitle={Loi binomiale}] Trois personnes s'apprêtent à passer le portique de sécurité. On suppose que pour chaque personne la probabilité que le portique sonne est égale à $0.14$. Soit $X$ la variable aléatoire donnant le nombre de personnes faisant sonner le portique, parmi les 3 personnes de ce groupe. \begin{enumerate} \item Tracer l'arbre représentant le situation. \item Justifier que $X$ suit une loi binomiale dont on précisera les paramètres. \item Quelle est la probabilité qu'une seule personne fasse sonner le portique? \item Calculer puis interpréter les probabilités suivantes \[ P(X = 0) \qquad \qquad P(X \geq 2) \] \item Calculer l'espérance de $X$ et interpréter le résultat. \end{enumerate} \end{exercise} \begin{solution} \begin{enumerate} \item \begin{tikzpicture}[sloped] \node {.} child {node {$0$} child {node {$0$} child {node {$0$} edge from parent node[above] {0.86} } child {node {$1$} edge from parent node[above] {0.14} } edge from parent node[above] {0.86} } child[missing] {} child {node {$1$} child {node {$0$} edge from parent node[above] {0.86} } child {node {$1$} edge from parent node[above] {0.14} } edge from parent node[above] {0.86} } edge from parent node[above] {0.86} } child[missing] {} child[missing] {} child[missing] {} child { node {$1$} child {node {$0$} child {node {$0$} edge from parent node[above] {0.86} } child {node {$1$} edge from parent node[above] {0.14} } edge from parent node[above] {0.86} } child[missing] {} child {node {$1$} child {node {$0$} edge from parent node[above] {0.86} } child {node {$1$} edge from parent node[above] {0.14} } edge from parent node[above] {0.86} } edge from parent node[above] {0.14} } ; \end{tikzpicture} \item Chaque personne a 2 possibilités (1: fait sonner ou 2: ne fait pas sonner) et l'on fait passer 3 personnes ce qui correspond à une répétition identique et aléatoire. On peut donc modéliser la situation par une loi binomiale. \[ X \sim \mathcal{B}(3; 0.76) \] \item Probabilité qu'une seule personne fasse sonner le portique. On voit qu'il y a 3 branches qui correspondent à cette situation dont \[ P(X = 1) = 3 \times 0.14^1 \times 0.86^2 \approx 0.311 \] \item \[ P(X = 0) = 0.86^3 \approx 0.636 \] \[ P(X \geq 2) = P(X = 2) + P(X = 3) = 3 \times 0.14^2 \times 0.86^1 + 0.14^3 \approx 0.054 \] \item Il faut d'abord tracer le tableau résumant la loi de probabilité: \begin{center} \begin{tabular}{|c|*{4}{c|}} \hline Valeur & 0 & 1 & 2 & 3 \\ \hline Probabilité & $0.636$ & $0.311$ & $0.051$ &$0.003$ \\ \hline \end{tabular} \end{center} On peut alors calculer l'espérance \[ E[X] = 0 \times 0.636 + 1 \times 0.311 + 2 \times 0.051 + 3 \times 0.003 = 0.42 \] On peut donc estimer qu'il y aura en moyenne $0.42$ personnes qui feront sonner le portique sur les 3 personnes. \end{enumerate} \end{solution} \begin{exercise}[subtitle={Équation puissance}] Résoudre les équations et inéquations suivantes \begin{multicols}{2} \begin{enumerate} \item $10^x = 5$ \item $20^x = 40$ \item $0.85^x \leq 38$ \item $2 \times 0.24^x = 24$ \end{enumerate} \end{multicols} \end{exercise} \begin{solution} Les solutions ci-dessous ne sont pas justifiée car l'ordinateur ne sait pas faire. Par contre, vous vous devez savoir justifier vos réponses! \begin{enumerate} \item $x = \log(5)$ \item $x = \frac{\log(40)}{\log(20)}$ \item Il faut faire attention quand on divise par un log car ce dernier peut être négatif ce qui est le cas ici. Il faut donc pense à changer le sens de l'inégalité. $x \geq \frac{\log(38)}{\log(0.85)}$ \item Il faut penser à faire la division à par $2$ avant d'utiliser le log car sinon, on ne peut pas utiliser la formule $\log(a^n) = n\times \log(a)$. $x = \frac{\log(12.0)}{\log(0.24)}$ \end{enumerate} \end{solution} \begin{exercise}[subtitle={Étude de fonctions}] Soit $f(x) = - 3x^3 + 49.5x^2 + 5580x - 3$ une fonction définie sur $\R$. \begin{enumerate} \item Calculer $f'(x)$ la dérivée de $f(x)$. \item Calculer $f'(31)$ et $f'(-20)$. \item En déduire une forme factorisée de $f'(x)$. \item Étudier le signe de $f'(x)$ et en déduire les variations de $f(x)$. \item Est-ce que la fonction $f(x)$ admet un maximum ou un minimum? Si oui, calculer sa valeur. \end{enumerate} \end{exercise} \begin{solution} \begin{enumerate} \item Dérivée de $f(x)$: $f'(x) = - 9x^2 + 99x + 5580$ \item \begin{align*} f'(31) &= - 9 \times 31^{2} + 99 \times 31 + 5580\\&= - 9 \times 961 + 3069 + 5580\\&= - 8649 + 8649\\&= 0 \end{align*} \begin{align*} f'(-20) &= - 9 \times - 20^{2} + 99(- 20) + 5580\\&= - 9 \times 400 - 1980 + 5580\\&= - 3600 + 3600\\&= 0 \end{align*} Donc $x = 31$ et $x=-20$ sont des racines de $f'(x) = - 9x^2 + 99x + 5580$. \item On en déduit la forme factorisée suivante \[ f'(x) = -9 (x - 31)(x--20) \] \item Pas de correction disponible \item À causes des branches extérieurs, la fonction $f(x)$ n'a pas de maximum ou de minimum. \end{enumerate} \end{solution} %\printsolutionstype{exercise} \end{document} %%% Local Variables: %%% mode: latex %%% TeX-master: "master" %%% End: